Can someone help!!! And explain please

Can Someone Help!!! And Explain Please

Answers

Answer 1

Answer:

400(π+2) feet square

Step-by-step explanation:

let x be the diagonal of the cage=40√2 at the same time it is the radius of the circle ( the tiger can go in circle)

but since the cage is part of the circle and not full turn πr²/8

area of the circleπr²+ half area square

(π(40√2)²)/8 +40²/2

3200π/8+1600/2

400π+800

400(π+2) feet square

Can Someone Help!!! And Explain Please

Related Questions

At the Olympic games, many events have several rounds of competition. One of these events is the men's 100 100100-meter backstroke. The upper dot plot shows the times (in seconds) of the top 8 88 finishers in the final round of the 2012 20122012 Olympics. The lower dot plot shows the times of the same 8 88 swimmers, but in the semifinal round. Which pieces of information can be gathered from these dot plots? (Remember that lower swim times are faster.) Choose all answers that apply: Choose all answers that apply:

Answers

Answer:

The center of the semifinal round distribution is greater than the center of final round distribution.

The variability in the semifinal round distribution is less than variability in the final round distribution.

Step-by-step explanation:

The mean value of each distribution set is not calculates as the center of semifinal round distribution is greater than the final round distribution. MAD Mean Absolute Deviation is calculated from the dotted graph plot, the distribution of semifinal round is less spread out than the final round distribution.

Answer:

correct answer is None of the above i understood nothing the other person was trying to say...

Step-by-step explanation:

mark me brainliest please...

Find the distance across the lake. Assume the triangles are similar.
80 m
х
у
20 m
60 m

Answers

Answer:

a

Step-by-step explanation:

Answer:

A.   L = 240 m

Step-by-step explanation:

use similar triangle

L / 60 = 80 / 20

L = (80 * 60) / 20

L = 240 m


31. Each day, Talisa exercises by first
stretching and then swimming
some laps, as shown in the table.
Make a scatter plot of the total
time she exercises as a function
of the number of laps she swims.
Draw a trend line.

Answers

Answer:

Step-by-step explanation:

Given the following :

Laps - - - - - - - - 5 - - - 6 - - - 7 - - - 8 - - - 9

Total time - - - 25 - - 28 - - 29 - - 30 - - 32

Using online graphing tool:

The y - axis named dependent variable represents the total time taken.

The x-axis, represents the number of laps.

The equation of the trend line attached to the plot is in the form :

y = mx + c

y = 1.6x + 17.6

Where y = total time taken

x = number of laps

m = 1.6 = gradient of the line (change in y / change in x)

C = 17.6 = intercept (whee the trndline intersects the y-axis).

I need help on both answers. They’re different from my other problems so I’m kinda confused

Answers

i think it would be b but
i’m not sure about the other one

What is the rate of change and initial value for the linear relation that includes the points shown in the table?
ху
1 20
3 10
5 0
7 -10
Initial value: 20, rate of change: 10
Initial value: 30, rate of change: 10
Initial value: 25, rate of change: -5
Initial value: 20, rate of change: -10

Answers

Answer:

Initial Value: 25, Rate of change -5.

First. Lets find the rate of change.

y2-y1/x2-x1 = m

We have A(1,20) B(3,10)

10-20/3-1=-5

m=-5(Rate of change)

Now let's find the initial value using slope-point form.

y-y₁=m(x-x₁)

y-20=-5(x-1)

=-5x+5+20

=-5x+25

The initial value is the value of y when the value of x is equal to 0. (Also the Y-Intercept)

Initial Value = -5(0)+25

=25

Find the value of x. Round to the nearest tenth.Find the value of x. Round to the nearest tenth.

Answers

Answer:

x = 55.6

Step-by-step explanation:

In order to find the value of x we use sine

sin ∅ = opposite / hypotenuse

From the question

x is the hypotenuse

the opposite is 19

So we have

sin 20 = 19/x

x = 19/sin 20

x = 55.55

We have the final answer as

x = 55.6 to the nearest tenth

Hope this helps you

Answer:

x = 55.6

Step-by-step explanation:

HELP ASAP PLEASE!!!!!!!!!!!!!!!!!

Answers

Answer:

A

C

D

Step-by-step explanation:

√54 or√9 *√6 or √27 *√4

are equal to the answer.

You can do that by doing the square of outer number which is 3 which equals to 9 when squared and multiplying that with the number inside the square root.

LCM of x<sup>2</sup>+5x+6 and x<sup>2</sup>-x-6 is ………………………





Answers

Answer:

[tex] (x^2 - 9)(x + 2) [/tex]

Step-by-step explanation:

Given:

[tex] x^2 + 5x + 6 [/tex]

[tex] x^2 - x - 6 [/tex]

Required:

LCM of the polynomials

SOLUTION:

Step 1: Factorise each polynomial

[tex] x^2 + 5x + 6 [/tex]

[tex] x^2 + 3x + 2x + 6 [/tex]

[tex] (x^2 + 3x) + (2x + 6) [/tex]

[tex] x(x + 3) + 2(x + 3) [/tex]

[tex] (x + 2)(x + 3) [/tex]

[tex] x^2 - x - 6 [/tex]

[tex] x^2 - 3x +2x - 6 [/tex]

[tex] x(x - 3) + 2(x - 3) [/tex]

[tex] (x + 2)(x - 3) [/tex]

Step 2: find the product of each factor that is common in both polynomials.

We have the following,

[tex] x^2 + 5x + 6 = (x + 2)(x + 3) [/tex]

[tex] x^2 - x - 6 = (x + 2)(x - 3) [/tex]

The common factors would be: =>

[tex] (x + 2) [/tex] (this is common in both polynomials, so we would take just one of them as a factor.

[tex] (x + 3) [/tex] and,

[tex] (x - 3) [/tex]

Their product = [tex] (x - 3)(x + 3)(x +2) = (x^2 - 9)(x + 2) [/tex]

What is 12.5% of 72

Answers

Answer:

[tex]\boxed{9}[/tex]

Step-by-step explanation:

[tex]\sf of \ refers \ to \ multiplication.[/tex]

[tex]12.5\% \times 72[/tex]

[tex]\frac{12.5}{100} \times 72[/tex]

[tex]\sf Multiply.[/tex]

[tex]\frac{900}{100} =9[/tex]

I NEED YOUR HELP PLS

Answers

Answer:

For question 1 you can try dividing each of the value

For instance, you can divide 9 by 25 and see if you get a nice number

e.g. 1/8=0.125, numbers like these

For the second question, you can find the fraction by dividing 1000 starting with the decimal points

e.g 0.650, you would be plotting 650/1000 and you would simplify the fraction to the lowest value any value above the decimal point you can multiply by the denominator and add the nominator value to get your final answer.

Step-by-step explanation:

Answer:

Write the denominator in its prime factors. If the prime factorization of the denominator of a fraction has only factors of 2 and factors of 5, the decimal expression terminates.  If there is any prime factor in the denominator other than 2 or 5, then the decimal expression repeats.

example: 9/25

25 = 5*5, so it will be terminating

example: 7/12

12 = 3*2*2, which contains a 3, so it will be repeating.

The width of a rectangle measures (6.8d-4.2)(6.8d−4.2) centimeters, and its length measures (9.2d+8.7)(9.2d+8.7) centimeters. Which expression represents the perimeter, in centimeters, of the rectangle?

Answers

Answer:

The perimeter of the rectangle is represented by [tex]p = 32\cdot d + 9[/tex], measured in centimeters.

Step-by-step explanation:

The perimeter ([tex]p[/tex]) of a rectangle, measured in centimeters, is represented by this formula:

[tex]p = 2\cdot (w+l)[/tex]

Where [tex]w[/tex] and [tex]l[/tex] are width and length, measured in centimeters.

If [tex]w = 6.8\cdot d-4.2[/tex] and [tex]l = 9.2\cdot d+8.7[/tex], the expression that represents the perimeter is:

[tex]p = 2\cdot (16\cdot d +4.5)[/tex]

[tex]p = 32\cdot d + 9[/tex]

The perimeter of the rectangle is represented by [tex]p = 32\cdot d + 9[/tex], measured in centimeters.

Suppose that $9500 is placed in an account that pays 9% interest compounded each year.
Assume that no withdrawals are made from the account.
Follow the instructions below. Do not do any rounding.
(a) Find the amount in the account at the end of 1 year.
so
(b) Find the amount in the account at the end of 2 years.
$
?

Answers

Answer:

$11286.95 second year

$10335 first year

Step-by-step explanation:

9% of 9500 is 855, 9500 plus 855 = 10335. (first year)

9% of 10335 is 931.95, and 10335+931.95 is 11286.95. (second year)

The amount in the account at the end of 1 year  $10335 (first year)

The amount in the account at the end of 2 years $11286.95.

What is the compound interest?

Compound interest is when you earn interest on both the money you've saved and the interest you earn.

Formula:

A = P(1 + {r}/{n})^{n.t}

here, we have,

$9500 is placed in an account that pays 9% interest compounded each year.

so, we get,

9% of 9500 is 855,

9500 plus 855 = 10335. (first year)

again,

9% of 10335 is 931.95,

and 10335+931.95 is 11286.95. (second year)

Hence, The amount in the account at the end of 1 year  $10335 (first year)

The amount in the account at the end of 2 years $11286.95.

To learn more on Compound interest click:

brainly.com/question/29335425

#SPJ2

8 more than a number

Answers

Answer:

[tex]\boxed{ 8 + x}[/tex]

Step-by-step explanation:

Hey there!

In most cases "the number" would be x.

So if the statement says 8 "more than a number",

It is saying 8 plus x or 8 + x.

Hope this helps :)

Answer:

x + 8 is the meaning.

Step-by-step explanation:

“more” means addition. Take the number as “x”, so it will be x + 8.

That's the answer.

Two buildings are 12m apart on the same horizontal level. From the top of the taller building, the angle of depression of the bottom of the shorter building is 48degrees and from the bottom, the angle of of elevation of the top of the shorter building is 36 degrees. Calculate the difference in the heights of the buildings

Answers

Answer:

4.61 m

Step-by-step explanation:

The angle of depression of the bottom of the shorter building from the top of the taller building = 48° equals the angle of elevation of the top of the taller building from the bottom of the shorter building

Using trig ratios

tan48° = H/d where H = height of taller building and d = their distance apart = 12 m

H = dtan48° = 12tan48° = 13.33 m

Also, the angle of elevation of the top of the shorter building from the bottom of the taller building is 36°

Using trig ratios

tan36° = h/d where h = height of shorter building

h =dtan36° = 12tan36° = 8.72 m

Now, the difference in height of the buildings is thus H - h = 13.33 m - 8.72 m = 4.61 m

Complete the following two-way frequency table.

Answers

Answer:

Step-by-step explanation:

Number of candies with Forest = 12

Candies containing coconut and chocolate both = Number common in coconut and the chocolate = 3

Candies which do not contain coconut but contain the chocolate = 6

Candies which contain the coconut but do not contain the chocolate = 1

Candies which neither contain the chocolate nor coconut = 2

From the given Venn diagram,

                                                Contain coconut         Do not contain coconut

Contain chocolate                              3                                       6

Do not contain chocolate                 1                                        2

Find the missing probability. P(A)=1120,P(B|A)=1320,P(A∩B)=?

Answers

Answer: 143/400

Explanation:

Assuming you meant to say

P(A) = 11/20

P(B|A) = 13/20

then,

P(A∩B) = P(A)*P(B|A)

P(A∩B) = (11/20)*(13/20)

P(A∩B) = (11*13)/(20*20)

P(A∩B) = 143/400

THE ANSWER IS 143/400 !

Type the correct answer in the box. Use numerals instead of words. If necessary, use / for the fraction bar. Stacy goes to the county fair with her friends. The total cost of ride tickets is given by the equation c = 3.5t, where c is the total cost of tickets and t is the number of tickets. If Stacy bought 15 tickets, she would spend $

Answers

Answer:

$52.2

Step-by-step explanation:

Given her total cost of ride tickets modeled by the equation c = 3.5t where c is the total cost of tickets and t is the number of tickets, If Stacy bought 15 tickets, to know the amount she would spend on 15 tickets, we will substitute t = 15 into the modeled equation as shown;

[tex]c = 3.5t\\when t = 15\\\\c = 3.5(15)\\\\c = \frac{35}{10} * 15\\ \\c = \frac{5*7}{5*2} * 15\\\\[/tex]

[tex]c = \frac{7}{2} * 15\\ \\c = \frac{105}{2}\\ \\c = \ 52.2[/tex]

Hence Stacy would spend $52.2 on 15 tickets

Answer:

I hope this helps!

Step-by-step explanation:

|3x–1|=8 please help!!!!!

Answers

Answer: -3

Add 1 to both sides

[tex]3x-1+1=8+1[/tex]

[tex]3x=9[/tex]

Divide both sides by 3

[tex]3x/3=9/3\\x=3[/tex]

how many are 8 raised to 4 ???​

Answers

2 cause you divide it

Find the measure of each side indicated. Round to the nearest tenth.

A) 19.8
C) 24.9
B) 27.2
D) 25.3

Answers

Answer:

D. 25.3

Step-by-step explanation:

tan∅ = opposite over adjacent

Step 1: Write equation

tan66.5° = x/11

Step 2: Multiply both sides by 11

11tan66.5° = x

Step 3: Evaluate

x = 25.2983

x ≈ 25.3

Answer:

[tex]\huge\boxed{x = 25.3}[/tex]

Step-by-step explanation:

Tan θ = opposite / adjacent

Where θ = 66.5 , opposite = x and adjacent = 11

Tan 66.5 = x / 11

2.3 * 11 = x

25.3 = x

OR

x = 25.3

Evaluate a + b for a= 34 and b= -6

Answers

Answer:

Hey there!

a+b

34+(-6)

34-6

28

Let me know if this helps :)

Can you please Solve for x

Answers

x - 3 = 27

add three to both sides

then x= 30

●✴︎✴︎✴︎✴︎✴︎✴︎✴︎✴︎❀✴︎✴︎✴︎✴︎✴︎✴︎✴︎✴︎✴︎●

         Hi my lil bunny!

❧⎯⎯⎯⎯⎯⎯⎯⎯⎯⎯⎯⎯⎯⎯⎯⎯⎯⎯⎯⎯⎯⎯⎯⎯⎯⎯⎯⎯⎯⎯⎯⎯⎯⎯⎯⎯⎯⎯☙

Let's solve your equation step-by-step.

[tex]x-3=27[/tex]

Step 1: Add 3 to both sides.

[tex]x -3 + 3 = 27 +3[/tex]

[tex]x = 30[/tex]

So the answer is : [tex]x = 30[/tex]

❧⎯⎯⎯⎯⎯⎯⎯⎯⎯⎯⎯⎯⎯⎯⎯⎯⎯⎯⎯⎯⎯⎯⎯⎯⎯⎯⎯⎯⎯⎯⎯⎯⎯⎯⎯⎯⎯⎯☙

●✴︎✴︎✴︎✴︎✴︎✴︎✴︎✴︎❀✴︎✴︎✴︎✴︎✴︎✴︎✴︎✴︎✴︎●

Hope this helped you.

Could you maybe give brainliest..?

❀*May*❀

Reduce 5/15 to its lowest terms

Answers

Answer:

The answer is 1/3

Answer:

1/3

Step-by-step explanation:

The factors of 5 are 1,5;

* The factors of 15 are 1,3,5,15.

We can see that the GCD is 5 because it is the largest number by which 5 y 15 can be divided without leaving any residue.

To reduce this fraction, simply divide the numerator and denominator by 5 (the GCF).

So, 5 /15

= 5÷5 /15÷5

= 1 /3

Can someone please tell me how to solve this problem??!! I literally have to go back in math if I don’t pass this HELP!!

Answers

Answer:

          D.   270° < φ < 360°

Step-by-step explanation:

Imagine coordinate system

I quarter is where x>0 and y>0  {right top} and it is (0°,90°)

II quarter is where x<0 and y>0  {left top} and it is (90°,180°)

III quarter is where x<0 and y<0  {left bottom} and it is (180°,270°)

IV quarter is where x>0 and y<0  {right bottom} and it is (270°,360°)

Now, we have an angle wich vertex is point (0,0) and one of its sides is X-axis and the second lay at one of the quarters.

For the trig functons of an angle created by this second side always are true:

In first quarter all functions are >0

in second one only sine

in third one: tangent and cotangent

and in fourth one: cosine

{You can check this by selecting any point on the second side of angle and put it's coordinates to formulas of these functions:

[tex]\sin \phi=\dfrac y{\sqrt{x^2+y^2}}\,,\quad \cos \phi=\dfrac x{\sqrt{x^2+y^2}}\,,\quad \tan\phi=\dfrac yx\,,\quad \cot\phi=\dfrac xy[/tex]  }

So:

sinφ<0  ⇒ III or IV quarter

tanφ<0  ⇒ I or IV quarter

IV quarter  ⇒  φ ∈ (270°, 360°)

1: The best statement for reason 6 of this proof is -∠A ≅ ∠C
-∠B ≅ ∠D
-∠B and ∠D are supplements
-∠B ≅ ∠B

2.The best reason for statements 3.5. and 7 in this proof is
- Alternate interior angles are congruent.
-Corresponding angles are congruent.
-Alternate exterior angles are congruent.
-Interior angles on the same sides of a transversal are supplements.

3. The best statement for reason 8 of this proof is
-∠B ≅ ∠B -∠A and ∠C are supplements.
-∠B ≅ ∠D
-∠A ≅ ∠C

Answers

Answer:

1) -∠B ≅ ∠D

2) -Interior angles on the same side of a transversal are supplementary

3) -∠A ≅ ∠C

Step-by-step explanation:

1) Given that ∠A and ∠B are supplements and ∠A and ∠D are supplements, we have; ∠B ≅ ∠D

2)  Given that ABCD is a parallelogram, therefore ∠A and ∠B,  ∠A and ∠D and ∠B and ∠C  are interior angles on the same side of a transversal and are therefore supplementary

3) Given that ∠A and ∠B and ∠B and ∠C are supplementary, therefore, ∠A ≅ ∠C.

Greyson completes a dive from a
cliff 75-feet above a river. It takes
him only 1.5 seconds to hit the
water and then another 0.5
second to descend 10 feet into the river

what’s the x axis and y axis?

Answers

Answer: y: height, x: time.

Step-by-step explanation:

The data we have is:

The initial position of Greyson is 75ft above the river.

He needs 1.5 seconds to hit the water, and other 0.5s tho reach the bottom of the river.

Then we have a relationship of height vs time.

The y axis will represent the heigth of Greyson, and the x-axis will represent the time, such that at the time x = 0 seconds, we have y = 75ft

1. In a right triangle, the lengths of the legs are a and b. Find the length of a hypotenuse, if: a=1, b=1; 2. In a right triangle, the length of a hypotenuse is c and the length of one leg is a. Find the length of the other leg, if: c=5, a=3;

Answers

Answer:

1. [tex]c = \sqrt{2}[/tex].

2. b = 4.

Step-by-step explanation:

To solve these two questions, keep the Pythagorean Theorem in mind: [tex]a^2 + b^2 = c^2[/tex]. Also remember that measurements cannot be negative, so we will disregard the negative answers.

1. a = 1, and b = 1. c = ?

[tex]1^2 + 1^2 = c^2[/tex]

[tex]1 + 1 = c^2\\[/tex]

[tex]c^2 = 1 + 1\\c^2 = 2\\\sqrt{c^2} = \sqrt{2}\\c = \sqrt{2}[/tex]

2. a = 3, c = 5. b = ?

[tex]3^2 + b^2 = 5^2\\9 + b^2 = 25\\b^2 = 16\\\sqrt{b^2} = \sqrt{16}\\b = 4[/tex]

Hope this helps!

A shop sells DVDs and CDs.

DVDs are sold at one price.
CDs are sold at a different price.

2 DVDs and 1 CD cost £35
2 DVDs and 2 CDs cost £45

Martin has £50 Does he have enough to buy 1 DVD and 3 CDs?

Answers

Answer:

Step-by-step explanation:

Lets Price of a DVD is fixed i.e. 15

and One CD price is 5 (Not fixed)

In First situation

2 DVDs and 1 CD cost = 35 as given

2 x 15 + 5 = 35

Lets one CD price is 7.5

In Second situation

2 x 15 + 2 x 7.5 = 45

Its mean CD price may be between 5 to 7.5

In asked scenario, Martin has 50

1 DVD and 3 CDs?

1 x 15 + 3 x 7.5 = 37.5

37.5 is lesser than 50

Hence Martin has enough to buy 1 DVD and 3 CDs.

The cost of milk is modeled by a linear equation where four quarts (one gallon) costs $3.09 while two quarts
(half-gallon) costs $1.65. Write the linear equation that expresses the price in terms of quarts. How much would
an eight-quart container of milk cost?

Answers

Answer:

linear equation to express the price is:

y=0.72x+0.21

An eight quarts will cost :  $5.97

Step-by-step explanation:

linear equation represent y=mx+b

let x=quarts ( x=4, x=2)

y= price (3.09 and y=1.65 )

two points (4,3.09) and (2,1.65)

need to find the slope m:

y2-y1/x2-x1

(1.65-3.09)/(2-4) ⇒ m=0.72

y=0.72x+b  find b at point (2,1.65)

1.65=0.72(2) +b  ⇒ b=0.21

y=0.72x +0.21

check : point (4,3.09)

y=0.72(4) +0.21

y=3.09  ( correct)

An eight quarts will cost :

y=0.72(8)+0.21

y=5.97 dollars

1)Sheyna drive to the lake and back. It took two hours less time to get there than it did to get back. The average speed on the trip there was 60 mph. The average speed on the way back was 36 mph. How many hours did the trip there take?

Answers

Answer:

8 hours

Step-by-step explanation:

Given:

Sheyna drives to the lake with average speed of 60 mph and

[tex]v_1 = 60\ mph[/tex]

Sheyna drives back from the lake with average speed of 36 mph

[tex]v_2 = 36\ mph[/tex]

It took 2 hours less time to get there than it did to get back.

Let [tex]t_1[/tex] be the time taken to drive to lake.

Let [tex]t_2[/tex] be the time taken to drive back from lake.

[tex]t_2-t_1 = 2[/tex] hrs ..... (1)

To find:

Total time taken = ?

[tex]t_1+t_2 = ?[/tex]

Solution:

Let D be the distance to lake.

Formula for time is given as:

[tex]Time =\dfrac{Distance}{Speed }[/tex]

[tex]t_1 = \dfrac{D}{60}\ hrs[/tex]

[tex]t_2 = \dfrac{D}{36}\ hrs[/tex]

Putting in equation (1):

[tex]\dfrac{D}{36}-\dfrac{D}{60} = 2\\\Rightarrow \dfrac{5D-3D}{180} = 2\\\Rightarrow \dfrac{2D}{180} = 2\\\Rightarrow D = 180\ miles[/tex]

So,

[tex]t_1 = \dfrac{180}{60}\ hrs = 3 \ hrs[/tex]

[tex]t_2 = \dfrac{180}{36}\ hrs = 5\ hrs[/tex]

So, the answer is:

[tex]t_1+t_2 = \bold{8\ hrs}[/tex]

Other Questions
On a coordinate plane, 2 lines are shown. Line A B has points (negative 4, negative 2) and (4, 4). Line C D has points (0, negative 3) and (4, 0). Which statement best explains the relationship between lines AB and CD? They are parallel because their slopes are equal. They are parallel because their slopes are negative reciprocals. They are not parallel because their slopes are not equal. They are not parallel because their slopes are negative reciprocals. g One of the harmonics in an open-closed tube has frequency of 500 Hz. The next harmonic has a frequency of 700 Hz. Assume that the speed of sound in this problem is 340 m/s. a. What is the length of the tube What is each half of a chromosome called?O A. DemisomeOB. Spindle FiberC. ChromatidD. Centromere HIV is a disease that reduces T cell production in the immune system. Would HIV increase or decrease a person's chance of developing cancer cells? a. The person's chances would increase because T cells can destroy cancer cells. b. The person's chances would increase because T cells prevent cancer cell division. c. The person's chances would decrease because T cells create lymph, which reduces cancer cell growth. d. The person's chances would decrease because T cells are especially susceptible to becoming cancerous. 4.Aliyah, Brenda and Candy share a sum of money in the ratio of 3:5:6. AfterCandy gives $100 to Aliyah and $50 to Brenda, the ratio becomes 2 : 3:3.(a) Suppose Aliyah has $3x at the start, express Candy's initial sum of money interms of x.(b) Find the value of x.(c) Hence, how much money does Brenda have in the end? you move left 2 units and right 9 units. you end at (4,5). where did you start? A particle moves according to a law of motion s = f(t), t 0, where t is measured in seconds and s in feet. (If an answer does not exist, enter DNE.) f(t) = t3 8t2 + 27t Transform the polar equation to a Cartesian (rectangular) equation: r= 4sinoptions include: x^2+y^2 = 4yx^2+y^2 = -4x^2+y^2 = 4x^2+y^2 = -4y The economy in the Southern colonies was based on What do nitrifying bacteria do? 5) What are the states annexed by Doctrine of lapse? find the multiplicative inverse of 3 by 4 minus 5 by 7 Find the sum of (5x3 + 3x2 - 5x + 4) and (8x3 -5x2 + 8x + 9) help...help...asap... hat a 15 kg body is pulled along a horizontal fictional table by a force of 4N what is the acceleration of the body The pepper plant has 2/3 as many fruits on it as the tomato plant has. The tomato plant has 9 fruits on it. How many fruits does the pepper plant have on it? What is 5 feet and 11 inches in inches 14. Twice the sum of a number and eight How can you identify Bangladesh in a unique way in the world map ? Ramesh or Harish______won the prize. ( verb has/ have)